Berechnung der Verschiebungsamplitude des Ultraschall-Leistungswandlers

Ich bin kein Physiker, aber mein aktuelles Projekt treibt mich zu einigen physikbezogenen Berechnungen und suche daher Hilfe.

Ich habe einige Ultraschallwandler, 5938D-25LBPZT , für die nur sehr begrenzte Informationen verfügbar sind.

Transducer-Diagramm Wandler-Foto

Um die Parameter für einen Ultraschallverstärker und ein Verstärkungshorn für dieses Gerät zu definieren, muss ich die Verschiebungsamplitude seiner strahlenden Oberfläche berechnen. Das Verfahren, dem ich folge, und die Basisdaten ( oder, wenn Daten nicht verfügbar sind, meine Annahmen ) lauten wie folgt:

Betriebsleistung: 50   W RMS (für den der Wandler ausgelegt ist 60   W )
Material des emittierenden Horns: Edelstahl SS316 (beste Vermutung, leider, da SS316 typisch für andere derartige Wandler ist)
Durchmesser der emittierenden Fläche: 5.865   C M (gemessen)
Frequenz bei Resonanz: F = 24 989   H z (gemessen)
Optimale Kraftübertragung: 90   % (in Vorwärtsrichtung, nach verschiedenen Quellen)

Konstanten :

Dichte von SS316: ρ = 7.8   G / C M 3
Geschwindigkeit der Schallausbreitung in Längsrichtung: C = 5.8   M M / μ S (aus dieser Referenz , ca.)
Sollte stattdessen die axiale Schallgeschwindigkeit für diese Berechnung berücksichtigt werden?

Berechnungen bisher :

Gesichtsbereich: 27.016   C M 2 (ohne Berücksichtigung des durch das Bolzenloch verlorenen Bereichs)
Schallintensität an der Stirnseite: ICH = 90   % × Leistung / Bereich = 1.666   W / C M 2
Verschiebungsamplitude A :

A = 2 ICH ( ρ C ) ω 2
Wo ω = 2 × π × F
Daher, A = 172.8296514   N M


Meine Fragen sind:

  1. Ich bin mir nicht sicher, ob ich es geschafft habe, alles in konsistente SI-Einheiten umzuwandeln, also brauche ich Hilfe, um zu bestätigen, dass ich diesen Teil richtig gemacht habe
  2. Wenn jemand Erfahrung mit solchen Leistungs-Ultraschall-Piezowandlern hat, wäre eine Bestätigung, dass meine Ergebnisse mehr oder weniger im Bereich der Plausibilität liegen, sehr hilfreich.
  3. Gibt es eine signifikant falsche Annahme oder einen konstanten Wert im Vorhergehenden?
  4. Sollte ich eine Online-Ressource verwenden, die all dies trivial aus der Nennleistung und anderen Eingaben berechnet, anstatt manuell damit zu kämpfen?

Update : Hinzufügen von Diagrammen, um die Anregungsrichtung und die beteiligten Materialien anzuzeigen.

Ultraschallstapel
(Bitte Diamanteinsatz ignorieren )

Dies ist die endgültige Anordnung des Ultraschallstapels. Der Booster und das Horn im obigen Diagramm sind beide SS316. Um die Maße dieser beiden Teile für die Resonanz zu berechnen, benötige ich die Auslenkungsamplitude der Stirnfläche des stählernen "Strahlkopfes" im Wandler ( erstes Diagramm hier ), der mit dem Booster verschraubt werden soll. Das Bolzenloch im Strahlungskopf ist auf dem Foto oben zu sehen.

Ein weiteres Diagramm für einen ähnlichen "Ultraschallstapel", bei dem der Langevin-Wandler selbst aufgeteilt ist. Dieses Diagramm hat einen zylindrischen Strahlungskopf, im Gegensatz zu der Kegelschnittform in den Wandlern, für die ich berechnen muss.

Ultraschallstapel
(Bitte ignorieren Sie Wedge )

(Die letzten beiden Diagramme stammen aus Artikeln auf ScienceDirect.com )

Antworten (1)

  1. Deine Berechnungen scheinen richtig zu sein. Ich glaube, die Dichte in Ihrer Gleichung entspricht der Dichte des Materials, durch das sich Ihre Schallwelle ausbreitet. Ihre Verwendung von Stahl zeigt an, dass Sie Schall durch eine Stahlplatte (nicht Luft) senden. Wenn dies beabsichtigt ist, sind Ihre Berechnungen korrekt.
    Sie sollten die axiale Schallgeschwindigkeit verwenden, wenn Ihr Wandler senkrecht zur Übertragungsrichtung schwingt. Dies würde beispielsweise gelten, wenn Sie einen Wandler und einen Empfänger mit parallel zueinander ausgerichteten Sendeachsen auf der gleichen Seite einer langen Platte platziert hätten. Wenn der Wandler dagegen parallel zur Übertragungsrichtung schwingt (z. B. am Ende einer Stahlstange, entlang der Stange übertragend), dann ist die Längsschallgeschwindigkeit die geeignete Geschwindigkeit. (Wenn Sie beabsichtigen, durch Luft zu übertragen, werden Scherwellen nicht unterstützt. Außerdem ist die von Ihnen verwendete Gleichung im Fall von Luft nicht vollständig anwendbar, da das emittierende Horn eher ein endlicher Körper als ein quasi unendliches Medium ist wie von der Gleichung angenommen.)

  2. Der von Ihnen berechnete Wert ist angemessen. Es gibt eine Reihe von (durch Paywall blockierten) wissenschaftlichen Artikeln, die sich mit Wandleramplituden befassen. Diese Amplituden reichen von minimal nachweisbaren Werten von etwas über 1 Angström (0,1 nm) bis hin zu bruchauslösenden Amplituden von ~5 Mikron (5000 nm). Natürlich hängen diese Werte stark von der Antriebsfrequenz, der Leistung und der Geometrie des Wandlers ab. Sie sollten jedoch Vertrauen in die von Ihnen berechneten Werte haben. Ich würde hinzufügen, dass, da die Unsicherheit in Ihrer Materialdichte und Schallgeschwindigkeit etwas groß ist, auch die Unsicherheit in Ihrer Amplitude sein wird. Um diese Unsicherheit genauer widerzuspiegeln (unter Verwendung nur signifikanter Zahlen und nicht einer streng berechneten Unsicherheit), ist Ihre Amplitude ungefähr 1.7 × 10 5 cm.

Bearbeiten als Antwort auf den Kommentar: Ich verstehe jetzt besser, was Sie tun. Die oben verwendete Gleichung dient zur Berechnung der Amplitude von Schallwellen, die sich durch ein einheitliches Medium mit bekannter Dichte und Steifigkeit (äquivalent bekannter Dichte und Schallgeschwindigkeit) ausbreiten. In der von Ihnen angelegten Geometrie möchten Sie die Schallgeschwindigkeit in Längsrichtung, da die Treiberachse parallel zum Horn verläuft, das der Sendekörper ist. Allerdings muss ich zugeben, dass die Verwendung eines kleinen Boosters und Horns das obige Ergebnis ungültig macht. Das Ergebnis, das Sie berechnet haben, ist die Amplitude der Schwingungen, die bei einem bekannten Kraftfluss und einer bekannten Frequenz in einem unendlichen Material angetrieben würden. Diese Art der Analyse wäre ziemlich gut geeignet, um Signale durch Träger in Brücken zu senden und Fehler zu finden, wenn Anomalien im erwarteten Ergebnis gefunden werden. In der von Ihnen beschriebenen Anordnung besteht der eigentliche Sinn des Boosters darin, diese Reaktion zu unterdrücken, indem eine ungleichmäßige Massenverteilung erzeugt wird, wodurch die Amplitude des Wandlers modifiziert wird. Wenn Sie versuchen, den erforderlichen Booster und die benötigte Sonotrode (Horn) zu berechnen, müssen Sie die Herstellerspezifikationen konsultieren, um zu erfahren, wie sie die Treiberamplitude definieren. Es scheint mir, dass Booster normalerweise auf zwei Parameter ausgelegt sind - die Verstärkung und die Frequenz. Der Wandler hingegen gibt in erster Linie Frequenz und Leistung vor. Die Amplitude hängt stark vom Rest des Systems ab, dh von der Masse des Horns, und womit auch immer der Keil in Kontakt kommt. Beim Versuch, mehr über das Thema herauszufinden, glaube ich, dass ich es gefunden habeeine Antwort auf Ihre Frage Nummer 4 , obwohl ich nicht sicher bin, wie viel es Ihnen nützen wird. Es genügt zu sagen, dass diese Art von Problemen durch die Finite-Elemente-Analyse behandelt wird, nicht durch die einfache und elegante, aber sehr begrenzte Gleichung, die Sie oben verwendet haben.

Vielen Dank für diese Antwort. Ja, das Ausbreitungsmedium ist Stahl, bitte sehen Sie sich die neuen Diagramme und Erklärungen an, die ich der Frage hinzugefügt habe. Über die axiale v / s-Längsgeschwindigkeit weiß ich es nicht wirklich, da einige Berechnungen, die ich gefunden habe (z. B. für Ultraschallverstärker ) , axial verwenden, andere längs. Die Übertragung erfolgt geradlinig von der Elektrode entlang des Strahlkopfes durch den Booster bis zur Hornspitze . Im Moment sind meine Berechnungsanforderungen für die Strahlungskopfoberfläche.
@AnindoGhosh: Siehe aktualisierte Antwort.
Genau dieser Sonotrodenrechner (und seine erweiterte herunterladbare Version) löste meine Frage aus: Dieses Tool benötigt die unbelastete Verschiebungsamplitude des Strahlkopfs, um die Sonotrodenabmessungen zu berechnen.
Wenn sie wirklich unbelastet meinen, dann würde ich (naiv) annehmen, dass sie die Verdrängung in Luft meinen. Es sieht so aus, als ob ihr Rechner eher eine Eingangsamplitude als eine Eingangsleistung annimmt. In diesem Fall können Sie erwägen, die Amplitude anzugeben, die Sie erreichen möchten, und dann die Berechnung durchführen. Wenn Sie einen Booster verwenden möchten, ändert dieser die Amplitude und erhöht die Leistungsaufnahme des Wandlers. Die von Ihnen berechnete Amplitude mag ein guter Ausgangspunkt sein, aber das ist wirklich eine Frage für einen Ingenieur.
Vielen Dank für die bereits gelieferten Einblicke. Der Grund, warum ich mittendrin bin, ist, dass das beteiligte Ingenieurbüro nicht funktionale Abmessungen geliefert hat und der Kunde dachte, dass ein Elektronikingenieur so ziemlich wie jeder andere ist, und da war ich :-)
Hah ... ja, Engineering von dem, der am nächsten ist. Viel Glück.